2

Let $ \pi: H \to GL(V) $ be an irreducible representation. Define $ N_\pi^r(H) $ inductively by $$ N_\pi^{r+1}(H)=\{ g \in GL(V): g \pi(H) g^{-1} \subset N_\pi^r(H) \} $$ starting from $ N_\pi^1(H):= \pi(H) $.

Note that the set $ N_\pi^r(H) $ is generally not a group.

My question: Is $ N_\pi^r(H) $ closed under inverses?

Some thoughts: If $ \pi(H) $ is normal in $ GL(V) $ then $ N_\pi^r(H)=GL(V) $ for all $ r \geq 2 $ and the whole construction is trivial. So the question is only interesting if $ H $ is not a normal subgroup. It is certainly true for $ r=1 $ since groups are always closed under inverses.

Context:

This construction is used for a naturally occurring object called the Clifford hierarchy which appears in quantum computing. For example, the (single qubit) Clifford hierarchy is the case where $ V=\mathbb{C}^2 $ and $ H=\pi(H)= \left\langle \begin{bmatrix} 0 & 1 \\ 1 & 0 \end{bmatrix}, \begin{bmatrix} 1 & 0 \\ 0 & -1 \end{bmatrix} \right\rangle $ is the tautological representation of the single qubit Pauli group.

This is a follow-up question to Is this interesting set, defined in terms of conjugation, closed under inverses? which deals with the case of general groups, not representations.

  • Good point, your comment got me thinking so I think I wrote them all down here https://math.stackexchange.com/questions/4811640/what-are-the-normal-subgroups-of-gln-f – Ian Gershon Teixeira Nov 21 '23 at 16:14

1 Answers1

2

Maybe it is true under some restrictions on $\pi(H)$ but not in general. Here is an example. Consider $V={\mathbb R}^2$, $\pi(H)$ generated by matrices $$ \gamma_n=\left[\begin{array}{cc} 1+4n& -16 n^2\\ 1& -4n+1\end{array} \right] $$ where $n=0, 1, 2, 3,...$. The subgroup generated by these matrices is discrete, free of infinite rank, defines an irreducible faithful representation of $H=F_\infty$ to $GL(2, {\mathbb R})$. The matrix $$ \alpha= \left[\begin{array}{cc} 1& 4\\ 0& 1\end{array} \right] $$ belongs to $N^2_\pi(H)$, but $\alpha^{-1}\notin N^2_\pi(H)$. (One needs some hyperbolic geometry to prove this.)

Moishe Kohan
  • 97,719
  • 1
    This is beautiful! I would of course enjoy more details about the hyperbolic geometry angle of this. In a strange coincide I was literally just texting my friend https://math.stackexchange.com/users/444923/geoffrey-sangston about you about 11 minutes before you posted this saying how good your answers are and speculating about your actual identity. Strange that the timing lined up like that! Thanks so much for all the amazing answers you post on MSE I think I can safely speak for Geoffrey when I say both of us have learned so much from you! And if you're ever in Maryland we'd love to meet you! – Ian Gershon Teixeira Dec 03 '23 at 19:52
  • also if you liked this question I'm pretty convinced its unsalvageable for infinite groups at this point but for irreducible finite groups I'm still holding out hope. The latest version of the question is here https://mathoverflow.net/questions/459505/is-this-set-defined-in-terms-of-an-irreducible-representation-closed-under-inv I'd love to hear any thoughts you have or see another cool geometry-inspired counter example – Ian Gershon Teixeira Dec 03 '23 at 19:53